ELMO Shortlist 2014
Algebra
4
Find all triples
of injective functions from the set of real numbers to itself satisfying

for all real numbers
and
. (We say a function
is injective if
for any distinct real numbers
and
.)
Proposed by Evan Chen


for all real numbers






Proposed by Evan Chen
5
Let
denote the set of nonzero reals. Find all functions
satisfying
for all
with
.
Proposed by Ryan Alweiss


![\[ f(x^2+y)+1=f(x^2+1)+\frac{f(xy)}{f(x)} \]](http://latex.artofproblemsolving.com/a/b/a/aba22852bc66dc7056e00f68437aa65f6c3b045b.png)


Proposed by Ryan Alweiss
7
Find all positive integers
with
such that the polynomial
in the
variables
,
,
,
is irreducible over the real numbers, i.e. it cannot be factored as the product of two nonconstant polynomials with real coefficients.
Proposed by Yang Liu


![\[ P(a_1, a_2, ..., a_n) = a_1^n+a_2^n + ... + a_n^n - n a_1 a_2 ... a_n \]](http://latex.artofproblemsolving.com/a/8/d/a8de9f925985591d6d69b099266a49dd1c5c7b9f.png)





Proposed by Yang Liu
Combinatorics
1
You have some cyan, magenta, and yellow beads on a non-reorientable circle, and you can perform only the following operations:
1. Move a cyan bead right (clockwise) past a yellow bead, and turn the yellow bead magenta.
2. Move a magenta bead left of a cyan bead, and insert a yellow bead left of where the magenta bead ends up.
3. Do either of the above, switching the roles of the words ``magenta'' and ``left'' with those of ``yellow'' and ``right'', respectively.
4. Pick any two disjoint consecutive pairs of beads, each either yellow-magenta or magenta-yellow, appearing somewhere in the circle, and swap the orders of each pair.
5. Remove four consecutive beads of one color.
Starting with the circle: ``yellow, yellow, magenta, magenta, cyan, cyan, cyan'', determine whether or not you can reach
a) ``yellow, magenta, yellow, magenta, cyan, cyan, cyan'',
b) ``cyan, yellow, cyan, magenta, cyan'',
c) ``magenta, magenta, cyan, cyan, cyan'',
d) ``yellow, cyan, cyan, cyan''.
Proposed by Sammy Luo
1. Move a cyan bead right (clockwise) past a yellow bead, and turn the yellow bead magenta.
2. Move a magenta bead left of a cyan bead, and insert a yellow bead left of where the magenta bead ends up.
3. Do either of the above, switching the roles of the words ``magenta'' and ``left'' with those of ``yellow'' and ``right'', respectively.
4. Pick any two disjoint consecutive pairs of beads, each either yellow-magenta or magenta-yellow, appearing somewhere in the circle, and swap the orders of each pair.
5. Remove four consecutive beads of one color.
Starting with the circle: ``yellow, yellow, magenta, magenta, cyan, cyan, cyan'', determine whether or not you can reach
a) ``yellow, magenta, yellow, magenta, cyan, cyan, cyan'',
b) ``cyan, yellow, cyan, magenta, cyan'',
c) ``magenta, magenta, cyan, cyan, cyan'',
d) ``yellow, cyan, cyan, cyan''.
Proposed by Sammy Luo
2
A
by
grid has some black squares filled. The filled black squares form one or more snakes on the plane, each of whose heads splits at some points but never comes back together. In other words, for every positive integer
greater than
, there do not exist pairwise distinct black squares
,
, \dots,
such that
and
share an edge for
(here
).
What is the maximum possible number of filled black squares?
Proposed by David Yang











What is the maximum possible number of filled black squares?
Proposed by David Yang
3
We say a finite set
of points in the plane is very if for every point
in
, there exists an inversion with center
mapping every point in
other than
to another point in
(possibly the same point).
(a) Fix an integer
. Prove that if
, then any line segment
contains a unique very set
of size
such that
.
(b) Find the largest possible size of a very set not contained in any line.
(Here, an inversion with center
and radius
sends every point
other than
to the point
along ray
such that
.)
Proposed by Sammy Luo







(a) Fix an integer






(b) Find the largest possible size of a very set not contained in any line.
(Here, an inversion with center







Proposed by Sammy Luo
4
Let
and
be positive integers. The game of Monis, a variant of Tetris, consists of a single column of red and blue blocks. If two blocks of the same color ever touch each other, they both vanish immediately. A red block falls onto the top of the column exactly once every
years, while a blue block falls exactly once every
years.
(a) Suppose that
and
are odd, and moreover the cycles are offset in such a way that no two blocks ever fall at exactly the same time. Consider a period of
years in which the column is initially empty. Determine, in terms of
and
, the number of blocks in the column at the end.
(b) Now suppose
and
are relatively prime and
is odd. At time
, the column is initially empty. Suppose a red block falls at times
years, while a blue block falls at times
years. Prove that at time
, the number of blocks in the column is
, where ![\[ S = \left\lfloor \frac{2r}{r+b} \right\rfloor
+ \left\lfloor \frac{4r}{r+b} \right\rfloor
+ ...
+ \left\lfloor \frac{(r+b-1)r}{r+b} \right\rfloor
. \]](//latex.artofproblemsolving.com/a/6/e/a6eb0a7fd467375d208b71e677208fc434ce6880.png)
Proposed by Sammy Luo




(a) Suppose that





(b) Now suppose








![\[ S = \left\lfloor \frac{2r}{r+b} \right\rfloor
+ \left\lfloor \frac{4r}{r+b} \right\rfloor
+ ...
+ \left\lfloor \frac{(r+b-1)r}{r+b} \right\rfloor
. \]](http://latex.artofproblemsolving.com/a/6/e/a6eb0a7fd467375d208b71e677208fc434ce6880.png)
Proposed by Sammy Luo
5
Let
be a positive integer. For any
, denote by
the number of permutations of
with exactly
disjoint cycles. (For example, if
then
since
,
,
are the only such permutations.) Evaluate
Proposed by Sammy Luo










![\[ a_n n^n + a_{n-1} n^{n-1} + \dots + a_1 n. \]](http://latex.artofproblemsolving.com/b/3/e/b3e91d5b24cb78a6992b766383ad5efa589c5c4d.png)
6
Let
be the function from
to
such that
and
otherwise. For each positive integer
, let
be the remainder when
is divided by
.
Finally, for each nonnegative integer
, let
denote the number of pairs
such that
.
Find a closed form for
.
Proposed by Bobby Shen







![\[ f_{m-1}(x,y) + \sum_{j=-1}^{1} \sum_{k=-1}^{1} f_{m-1}(x+j,y+k) \]](http://latex.artofproblemsolving.com/4/1/3/41370d1803ae6044345b23d0daf1cc0032cc97bc.png)

Finally, for each nonnegative integer




Find a closed form for

Proposed by Bobby Shen
Geometry
1
Let
be a triangle with symmedian point
. Select a point
on line
such that the lines
,
,
and
are the sides of a cyclic quadrilateral. Define
and
similarly. Prove that
,
, and
are collinear.
Proposed by Sammy Luo













Proposed by Sammy Luo
2









Proposed by Yang Liu
3
Let
be a cyclic
-gon. Prove that for every point
not the circumcenter of the
-gon, there exists a point
such that
is constant for
.
Proposed by Robin Park







Proposed by Robin Park
4
Let
be a quadrilateral inscribed in circle
. Define
,
,
,
,
, and
. Prove that
,
, and the
-symmedian are concurrent.
Proposed by Robin Park











Proposed by Robin Park
5
Let
be a point in the interior of an acute triangle
, and let
be its isogonal conjugate. Denote by
and
the circumcircles of triangles
and
, respectively. Suppose the circle with diameter
intersects
again at
, and line
intersects
again at
. Similarly, suppose the circle with diameter
intersects
again at
, and line
intersects
again at
.
Prove that lines
and
are parallel.
(Here, the points
and
are isogonal conjugates with respect to
if the internal angle bisectors of
,
, and
also bisect the angles
,
, and
, respectively. For example, the orthocenter is the isogonal conjugate of the circumcenter.)
Proposed by Sammy Luo



















Prove that lines


(Here, the points









Proposed by Sammy Luo
6
Let
be a cyclic quadrilateral with center
.
Suppose the circumcircles of triangles
and
meet again at
, while the circumcircles of triangles
and
meet again at
.
Let
denote the circle passing through
as well as the feet of the perpendiculars from
to
and
.
Define
analogously as the circle passing through
and the feet of the perpendiculars from
to
and
.
Show that the midpoint of
lies on the radical axis of
and
.
Proposed by Yang Liu


Suppose the circumcircles of triangles






Let





Define





Show that the midpoint of



Proposed by Yang Liu
7
Let
be a triangle inscribed in circle
with center
, let
be its
-mixtilinear incircle,
be its
-mixtilinear incircle,
be its
-mixtilinear incircle, and
be the radical center of
,
,
. Let
,
,
be the points at which
,
,
are tangent to
. Prove that
,
,
and
are concurrent.
Proposed by Robin Park
























Proposed by Robin Park
8
In triangle
with incenter
and circumcenter
, let
be the points of tangency of its circumcircle with its
-mixtilinear circles, respectively. Let
be the circle through
that is tangent to
at
, and define
similarly. Prove that
have a common point
other than
, and that
.
Proposed by Sammy Luo














Proposed by Sammy Luo
9
Let
be a point inside a triangle
such that
. Let the projections of
onto
,
, and
be
respectively. Let
be the circumcenter of
,
be the foot of the altitude from
to
,
be the midpoint of
, and
be the point such that
is a parallelogram. Show that
is similar to
.
Proposed by Sammy Luo



















Proposed by Sammy Luo
10
We are given triangles
and
such that
,
. Let the circumcenter of
be
, and let the circumcircle of
intersect
again at
respectively. Prove that the perpendiculars to
through
respectively intersect at a point
, and the lines
intersect at a point
, such that
are collinear.
Proposed by Sammy Luo















Proposed by Sammy Luo
11
Let
be a triangle with circumcenter
. Let
be a point inside
, so let the points
be on
respectively so that the Miquel point of
with respect to
is
. Let the reflections of
over the midpoints of the sides that they lie on be
. Let the Miquel point of
with respect to the triangle
be
. Show that
.
Proposed by Yang Liu















Proposed by Yang Liu
12
Let
in
, and let
be a point on segment
. The tangent at
to the circumcircle
of
hits
at
. The other tangent from
to
touches it at
, and
,
. Prove that
.
Proposed by David Stoner















Proposed by David Stoner
13
Let
be a nondegenerate acute triangle with circumcircle
and let its incircle
touch
at
respectively. Let
hit arcs
of
at
respectively, and let
be the points on
such that
. If
is the center of
, prove that
are collinear if and only if
.
Proposed by David Stoner
















Proposed by David Stoner
Number Theory
1
Does there exist a strictly increasing infinite sequence of perfect squares
such that for all
we have that
?
Proposed by Jesse Zhang



Proposed by Jesse Zhang
2
Define the Fibanocci sequence recursively by
,
and
for all
. Prove that for all integers
, there exists an integer
such that the sum of the digits of
when written in base
is greater than
.
Proposed by Ryan Alweiss









Proposed by Ryan Alweiss
3
Let
and
be fixed integers each at least
. Find the largest positive integer
for which there exists a polynomial
, of degree
and with rational coefficients, such that the following property holds: exactly one of
is an integer for each
.
Proposed by Michael Kural






![\[ \frac{P(k)}{t^k} \text{ and } \frac{P(k)}{t^{k+1}} \]](http://latex.artofproblemsolving.com/a/1/c/a1c40c43bb3628f0489094b7d2f17868e9a8791d.png)

Proposed by Michael Kural
4
Let
denote the set of positive integers, and for a function
, let
denote the function
applied
times. Call a function
saturated if
for every positive integer
. Find all positive integers
for which the following holds: every saturated function
satisfies
.
Proposed by Evan Chen






![\[ f^{f^{f(n)}(n)}(n) = n \]](http://latex.artofproblemsolving.com/3/c/c/3cc52c7e629952b5a16df301adc0f8ad30a228f4.png)




Proposed by Evan Chen
5
Define a beautiful number to be an integer of the form
, where
and
is a positive integer.
Prove that each integer greater than
can be expressed as the sum of pairwise distinct beautiful numbers.
Proposed by Matthew Babbitt



Prove that each integer greater than

Proposed by Matthew Babbitt
6
Show that the numerator of
is a multiple of
for any odd prime
.
Proposed by Yang Liu
![\[ \frac{2^{p-1}}{p+1} - \left(\sum_{k = 0}^{p-1}\frac{\binom{p-1}{k}}{(1-kp)^2}\right) \]](http://latex.artofproblemsolving.com/4/7/0/4706a0db8b4824bef682ac6fb26de24be6be4bc9.png)


Proposed by Yang Liu
7
Find all triples
of positive integers such that if
is not divisible by any prime less than
, then
divides
.
Proposed by Evan Chen





Proposed by Evan Chen
8
Let
denote the set of positive integers. Find all functions
such that:
(i) The greatest common divisor of the sequence
is
.
(ii) For all sufficiently large integers
, we have
and
for all positive integers
and
.
Proposed by Yang Liu


(i) The greatest common divisor of the sequence


(ii) For all sufficiently large integers


![\[ f(a)^n \mid f(a+b)^{a^{n-1}} - f(b)^{a^{n-1}} \]](http://latex.artofproblemsolving.com/9/8/a/98aef7f23c9cc0c86cdc7f6eede168ed17f40b19.png)


Proposed by Yang Liu
9
Let
be a positive integer and let
be any positive real. Prove that for all sufficiently large primes
with
, there exists an positive integer less than
which is not a
th power modulo
, where
is defined by
Proposed by Shashwat Kishore








![\[ \log r = \varepsilon - \frac{1}{\gcd(d,p-1)}. \]](http://latex.artofproblemsolving.com/0/5/c/05c244b560898e263a72f0a913a9a403b8129fd1.png)
10
Find all positive integer bases
so that the number
![\[ \frac{{\overbrace{11 \cdots 1}^{n-1 \ 1's}0\overbrace{77 \cdots 7}^{n-1\ 7's}8\overbrace{11 \cdots 1}^{n \ 1's}}_b}{3} \]](//latex.artofproblemsolving.com/a/9/e/a9e3867271af82255183c74fa2ea3fc2cb9ec96c.png)
is a perfect cube in base 10 for all sufficiently large positive integers
.
Proposed by Yang Liu

![\[ \frac{{\overbrace{11 \cdots 1}^{n-1 \ 1's}0\overbrace{77 \cdots 7}^{n-1\ 7's}8\overbrace{11 \cdots 1}^{n \ 1's}}_b}{3} \]](http://latex.artofproblemsolving.com/a/9/e/a9e3867271af82255183c74fa2ea3fc2cb9ec96c.png)
is a perfect cube in base 10 for all sufficiently large positive integers

Proposed by Yang Liu
11
Let
be a prime satisfying
, and let
be a positive integer. Define
![\[ f(x) = \frac{(x-1)^{p^n}-(x^{p^n}-1)}{p(x-1)}. \]](//latex.artofproblemsolving.com/e/6/0/e601e147c5356e05bc5f3bdd745b371e1b6a069c.png)
Find the largest positive integer
such that there exist polynomials
,
with integer coefficients and an integer
satisfying
.
Proposed by Victor Wang



![\[ f(x) = \frac{(x-1)^{p^n}-(x^{p^n}-1)}{p(x-1)}. \]](http://latex.artofproblemsolving.com/e/6/0/e601e147c5356e05bc5f3bdd745b371e1b6a069c.png)
Find the largest positive integer





Proposed by Victor Wang